simetría en física cuántica y momento angular

Tengo problemas para probar este argumento en la física cuántica:

En un estado estacionario (sin degeneración), si el sistema es simétrico en el tiempo pero no necesariamente simétrico bajo rotaciones, demuestre que el valor esperado del momento angular es cero.

Como entendí:

  • Estado estacionario: la función de onda es un vector propio del hamiltoniano.

  • La simetría en el tiempo significa que el hamiltoniano es constante.

  • Si el sistema no es necesariamente simétrico bajo rotaciones, entonces el hamiltoniano no necesariamente conmuta con el operador de momento angular.

Pero, ¿cómo puedo probar el argumento con eso? ¿Alguien puede ayudarme?

¡Gracias!

A menos que haya entendido mal la pregunta, no creo que pueda probar eso, porque esa afirmación es falsa (considere, por ejemplo, un estado estacionario del átomo de hidrógeno: allí, L z = metro 0 ).
Hola, no se mucho sobre el atomo de Hidrógeno, pero se me olvido mencionar que el estado estacionario no es degenerado, sirve?
Revisé nuevamente la pregunta y parece la pregunta original que copié de mi ejercicio... tal vez la pregunta esté mal...
Tengo la sensación de que la afirmación sigue siendo falsa, pero podría estar equivocado (y si consideramos una partícula libre, es decir, una onda plana: allí, los estados no son degenerados, pero el momento angular sigue siendo distinto de cero. ..)
hola Noam, estoy bastante seguro de que "tiempo simétrico" aquí significa "simétrico bajo inversión de tiempo". En el sentido que está considerando (el hamiltoniano no depende explícitamente del tiempo) la proposición es falsa, considere el hamiltoniano H = gramo S z . Escribí mi respuesta bajo esta suposición, avísame si no es lo que estabas buscando.
@AccidentalFourierTransform. Pero hay degeneración en un sistema de partículas libres: mi i k X y mi i k X tener la misma energía.

Respuestas (1)

Supongo que "simétrico en el tiempo" aquí significa que el sistema es invariante bajo la simetría de inversión en el tiempo :

T H T = H

Recuerde que el momento angular j se transforma bajo inversión de tiempo como

T j T = j .

Primero, como señala AccidentalFourierTransform, notemos que el estado en cuestión no puede ser un giro de medio entero, por la degeneración de Kramer (ver los comentarios a continuación).

Ahora, supongamos que | mi es un estado propio no degenerado de H . Desde H viaja con T tenemos:

H T | mi = T H | mi = mi T | mi .
Desde | mi no es degenerado, y T es antiunitario, esto quiere decir que:
T | mi = mi 2 i α | mi .
Utilizando el carácter antiunitario de T puede ver fácilmente que WLOG podemos poner α = 0 .

Ahora, usando T | mi = | mi , la antiunitaridad de T y la hermiticidad de j tenemos:

mi | j | mi = ( mi | T j T | mi ) = ( mi | j | mi ) = mi | j | mi
lo que implica:
mi | j | mi = 0.


En respuesta al comentario de Noam Chai:

  1. El hecho de que H T | mi = mi T | mi , junto con la suposición de que los valores propios mi no es degenerado, me permite concluir que T | mi = C | mi para algún número complejo C .
  2. Desde T es en particular una isometría, debemos tener | C | = 1 , entonces C = mi 2 i α por algún número real 2 α entre 0 y 2 π , decir (también α no depende del tiempo, ya que ni T ni | mi lo hace, por suposición). Ahora, multiplicando la ecuación T | mi = mi 2 i α | mi por mi i α , y utilizando el hecho de que T es antilineal, obtenemos:
    mi i α | mi = mi i α T | mi = T ( mi i α | mi ) ,
    de modo que el factor de fase pueda ser realmente absorbido en la definición de | mi .
  3. El proviene de la definición del adjunto de un operador antilineal. Permítanme cambiar a la notación espacial de Hilbert del matemático:
    gramo | F ( gramo , F ) .
    Esta notación es más clara cuando se trata de operadores antilineales. Ahora, dejando de lado los problemas de dominio (que de hecho no ocurren en el caso de T ), el adjunto de un operador antilineal A se define por la ecuación:
    ( A gramo , F ) = ( gramo , A F ) .
    Ves que el LHS es lineal en F , por lo que debe ser el RHS, por lo tanto, el . En esta notación, con | mi F , mi última ecuación dice:
    ( F , j F ) = ( T F , j T F ) = ( F , T j T F ) .
Si el sistema es simétrico bajo T , entonces no puede haber ningún estado no degenerado (según el teorema de Kramers )
El teorema de Kramers se aplica a giros de medio entero, ¿verdad? Además, no sé por qué hice tanto lío para demostrar que H T | mi = mi T | mi XDD
El teorema de Kramers es mucho más general: se aplica a cualquier sistema que sea invariante bajo T (La prueba general se puede encontrar en alguna parte del QFT de Weinberg, Vol. I). Inicialmente, también pensé que la simetría temporal del OP estaba relacionada con T , pero esto no puede ser, porque en tal caso no habría estados no degenerados.
En realidad, tienes razón: encontré la prueba (página 80) y, de hecho, solo es válida para sistemas con un giro total de medio entero.
Gracias ! Algunas preguntas: 1. ¿Dónde usaste HT|E⟩=ET|E⟩? No veo ningún uso de este hecho. 2. ¿Puedes explicar más sobre alfa? ¿Se supone que la potencia exponencial depende del tiempo? (el valor propio de T)
Gracias ! Algunas preguntas: 1. ¿Dónde usaste HT|E⟩=ET|E⟩? No veo ningún uso de este hecho. 2. ¿Puedes explicar más sobre alfa? y ¿se supone que la potencia exponencial depende del tiempo? (el valor propio de T) 3. ¿Puede aclarar: ⟨E|J|E⟩=(⟨E|T†JT|E⟩)∗? ¿Por qué es conjugado?
Hola @Noam Chai, mira la respuesta actualizada.